MCQs Flashcards

1
Q

Case 1:
A man aged 57 years complained of recurrent attacks of chest pain provoked by exercise and relieved by rest. Clinical examination and ECG confirmed the diagnosis of stable angina pectoris.

1- The following drug is selected to treat the acute attack of angina:

a. Ranolazine
b. Verapamil.
c. Trimetazidine.
d. lsosorbid dinitrate

A

d. lsosorbid dinitrate

How well did you know this?
1
Not at all
2
3
4
5
Perfectly
2
Q

2- To treat the acute attack of angina, the drug selected in Q-1 (lsosorbid dinitrat) is given by the following route:

a-Oral
B-Subcutaneous
C-Intramuscular
D-Sublingual

A

D-Sublingual

How well did you know this?
1
Not at all
2
3
4
5
Perfectly
3
Q

3- The drug chosen in Q-1 (lsosorbid dinitrat) relieves the angina pain by

A- Central analgesic effect.
B-Veno-dilatation and decreased preload.
C-Accumulation of cAMP.
D-Inhibiting all cardiac properties

A

B-Veno-dilatation and decreased preload.

How well did you know this?
1
Not at all
2
3
4
5
Perfectly
4
Q

4- The drug chosen in Q-1 (lsosorbid dinitrat) may produce all the following adverse effects EXCEPT:

a-Postural hypotension.
b-Headache.
C-Bradycardia.
D-Cutaneous flush.

A

C-Bradycardia.

How well did you know this?
1
Not at all
2
3
4
5
Perfectly
5
Q

5-The following drug can be prescribed orally for prophylaxis of angina pectoris

a-Verapamil
b-Salbutamol
C-Warfarin
D-Minoxidil

A

a-Verapamil

How well did you know this?
1
Not at all
2
3
4
5
Perfectly
6
Q

6- The following mechanisms are involved in the antianginal effect of the drug selected in Q-5 (Verapamil) EXCEPT:

a. Produces coronary vasodilatation.
b. Decreases the afterload.
c. Decreases oxygen demands by the heart.
d. opens K+ channels.

A

c. Decreases oxygen demands by the heart.

How well did you know this?
1
Not at all
2
3
4
5
Perfectly
7
Q

7- The drug selected in Q-5 (Verapamil) is especially useful in treatment of angina with co-existing:

A- Peptic ulcer.
B- Hypotension.
C- Atrial arrhythmia.
D- Heart failure.

A

C- Atrial arrhythmia.

How well did you know this?
1
Not at all
2
3
4
5
Perfectly
8
Q

8-The following adverse effects are expected with the use of the drug chosen in Q-5 (Verapamil) EXCEPT:

A-Constipation
B-Hypotension
C-Bradycardia
D-Bronchospasm

A

D-Bronchospasm

How well did you know this?
1
Not at all
2
3
4
5
Perfectly
9
Q

9-The drug selected in Q-5 (Verapamil) is contraindicated in:

A-Heart block
B-Peptic ulcer
C-Migraine headache
D-Epilepsy

A

A-Heart block

How well did you know this?
1
Not at all
2
3
4
5
Perfectly
10
Q

10- Despite proper patient compliance with the drug therapy chosen in Q-5 (Verapamil), still the patient complains of frequent angina attacks, what another drug would you like to add?

a.Propranolol
b.Hydrochlorothiazide
c.lsosorbide mononitrate
c.Diltiazem

A

c.lsosorbide mononitrate

How well did you know this?
1
Not at all
2
3
4
5
Perfectly
11
Q

11- On prescribing the drug selected in Q-10 (lsosorbide mononitrate), the following precautions should be considered EXCEPT:

a. Allow daily 8-10 hours drug-free period.
b. Never stop the drug therapy suddenly.
c. If a dose of the drug is missed, double the dose the next time.
d. Do not use after expiry date.
e. Avoid concurrent use of sildenafil,

A

c. If a dose of the drug is missed, double the dose the next time.

How well did you know this?
1
Not at all
2
3
4
5
Perfectly
12
Q

12- The following drug is used orally to prevent coronary thrombosis:

a. Paracetamol
b. Digoxin
c. Aspirin
d.Heparin

A

c. Aspirin

How well did you know this?
1
Not at all
2
3
4
5
Perfectly
13
Q

Case 1:
A 55-year-old diabetic woman has been recently diagnosed with exertional angina for which transdermal nitroglycerin were started. After 3 weeks of therapy, angina attacks were less frequent but not completely prevented and she complained of palpitation, flush and vertigo. Which of the following is an appropriate next therapeutic step for this patient?

A. Add another nitrate preparation
B. Add propranolol
C. Add nifedipine
D. Add diltiazem

A

D. Add diltiazem

How well did you know this?
1
Not at all
2
3
4
5
Perfectly
14
Q

Case 3:
A 46-year-old man complained to his physician of insomnia nightmares, fatigue, diminished libido, and blanching of the fingers when exposed to cold. The man, recently diagnosed with exertional angina had been taking an antianginal drug for 1 month. Which of the following drugs most likely caused the patient’s symptoms?

A, Propranolol
B. lsosorbide mononitrate
C. Nitroglycerin
D. Nifedipine

A

A, Propranolol

How well did you know this?
1
Not at all
2
3
4
5
Perfectly
15
Q

A 47-year-old man, recently diagnosed with exertional angina, had started a therapy with a transdermal nitroglycerin preparation 2 weeks previously, He carefully applied a new patch every morning immediately after removing the old one. Anginal attacks had disappeared completely during the first week of therapy but were back after. Which of the following best explains the reason for his anginal episodes?

A. Vasospastic angina complicating the exertional angina
B. Cellular tolerance to nitroglycerin
C. Increased metabolism of nitroglycerin
D. Insufficient original nitroglycerin dosage

A

B. Cellular tolerance to nitroglycerin

How well did you know this?
1
Not at all
2
3
4
5
Perfectly
16
Q

A 71-year-o1d male with history of chronic stable angina, presents to the primary care clinic complaining that the anginal attacks are in in severity and frequency. His current medications Include atenolol and
nitroglycerin. Further investigations confirmed the diagnosis of unstable Angina. Which of the following is/are the most appropriate addition to his antianginal therapy?

A. Nifedipine
B. LMW heparin SC 12/hrs.
C. Aspirin
D. All of the above

A

D. All of the above

How well did you know this?
1
Not at all
2
3
4
5
Perfectly
17
Q

Case 1:
A 57-year-old man complained of palpitations shortly after taking a
tablet of his prescribed medication. The man was recently diagnosed
with variant angina for which he had started therapy with nifedipine
4 days earlier. Which of the following actions is most likely caused
the patient’s symptoms?

A.Coronary vasodilation
B.Decreased total peripheral resistance
C.Increased venous return to the heart
D. Decreased myocardial contractility
E. Coronary steal phenomenon

A

B.Decreased total peripheral resistance

How well did you know this?
1
Not at all
2
3
4
5
Perfectly
18
Q

Case 2:
A 54-year-oldman has been diagnosed recently with variant angina. The patient had been sufferer from a cerebellar astrocytoma for 2 years and from a second-degree atrioventricular block for 1 year. Which of the following antianginal drugs would be appropriate for this patient?

A. lsosorbide mononitrate
B. Diltiazem
C. Verapamil
D. Nifedipine
E. Propranolol

A

D. Nifedipine

How well did you know this?
1
Not at all
2
3
4
5
Perfectly
19
Q

Case 1
J.O. is a 54-war-old male who suffers from crushing substernal chest pain and radiation to his left arm which started 45 min. earlier although he administered 3 tablets of SL NTG 5 min. apart. The ambulance team suspecting acute MI. All the following therapies should be considered in this patient while transported to hospital EXCEPT:

a) Reteplase
by Aspirin SD
c) Morphine IV infusion
d)Intranasal 02

A

a) Reteplase

How well did you know this?
1
Not at all
2
3
4
5
Perfectly
20
Q

2.Once J.O. is admitted to the ER, the following therapy should be added:

a) Heparin
by Nitroglycerine
c) Metoprolol
d) All the above

A

d) All the above

How well did you know this?
1
Not at all
2
3
4
5
Perfectly
21
Q

3.To achieve reperfusion of the infarcted area, the following therapy should be given to JO:

a) Enalapril
b) Alteplase
c) Metoprolol
d) Nifeclipine

A

b) Alteplase

How well did you know this?
1
Not at all
2
3
4
5
Perfectly
22
Q

Case 2:
A 77-year-old woman was brought to the emergency department with an Acute Myocardial Infarction (AMI). Six months ago, she suffered from an Ml and began taking propranolol, aspirin, lovastatin Her current medications also included captopril for hypertension. Two days ago, she became nauseous and vomited, and she stopped taking all her medications. The sudden withdrawal of which of the following drugs most likely triggered the recent MI?

A. Aspirin
B. Propranolol
C. Lovastatin
D. Captopril

A

B. Propranolol

How well did you know this?
1
Not at all
2
3
4
5
Perfectly
23
Q

A 48 year old man was brought to the emergency department with acute myocardial infarction. The man regularly used sildenafil in preparation for sexual intercourse because of an erectile dysfunction. He had recently been diagnosed with exertional angina and he had been taking an appropriate prescribed therapy. Which of the following drugs most likely caused the patients MI?

A. Propranolol
B. Nitroprusside
C. Nitroglycerin
D. Verapamil

A

C. Nitroglycerin

How well did you know this?
1
Not at all
2
3
4
5
Perfectly
24
Q

A 53 year old man was taken to the emergency department because of chest pain that apparently had been ongoing for over 5 hours. His vital signs were; BP 165/100 mm Hg, HR 50 bpm, respiratory rate 22/min. Physical examination revealed signs of severe pulmonary congestion, and an ECG was consistent with an anterior acute myocardial infarction. An appropriate therapy was instituted that included an intravenous infusion of which of the following drugs?

A. Epinephrine
B. Metoprolol
C. Nitroglycerin
D. Verapamil

A

C. Nitroglycerin

How well did you know this?
1
Not at all
2
3
4
5
Perfectly
25
Q

Which of the following drugs could be prescribed to help in controlling her cardiac condition?

a. Isosorbide di-nitrate
b. Verapamil
c. Captopril
d. Frusemide

A

c. Captopril

How well did you know this?
1
Not at all
2
3
4
5
Perfectly
26
Q

Based on recent medical evidence the patient’s cardiologist decided to give her a fixed drug combination that he believed would have a good impact on her prognosis because it enhances the action of endogenous vasodilators. Which of the following is the combination likely to be?

a. Perindopril/hydrochlorothiazide
b. Losartan/amiloride
c. Sacubitril/valsartan
d. Sacubitril/ramipril

A

c. Sacubitril/valsartan

How well did you know this?
1
Not at all
2
3
4
5
Perfectly
27
Q

What would happen if you added Captopril to this combo (Sacubitril/valsartan)?

a. Continue
b. Be discontinued
c. Have dose reduction
d. Have dose increase

A

c. Have dose reduction

How well did you know this?
1
Not at all
2
3
4
5
Perfectly
28
Q

The doctor decided to add another medicine to the patient’s regimen. The patient came back the next visit asking the doctor why he had prescribed for her an antidiabetic medication when in fact she was not diabetic.

Which of the following that medication be?
a. Empagliflozin
b. Metfromin
c. Insulin
d. Acarbose

A

a. Empagliflozin

How well did you know this?
1
Not at all
2
3
4
5
Perfectly
29
Q

A male patient with HF with reduced ejection fraction (HFrEF), is complaining of hypervolemia and edema, which of the following drugs is indicated:

a. Furosemide
b. Hydrocortisone
c. Hydralazine
d. Salbutamol

A

a. Furosemide

How well did you know this?
1
Not at all
2
3
4
5
Perfectly
30
Q

All of the following are the beneficial effects for adding diuretics in treatment of heart failure EXCEPT:

a. Improving congestive symptoms
b. Decreasing preload
c. Decreasing blood volume
d. Increasing cardiac output

A

d. Increasing cardiac output

How well did you know this?
1
Not at all
2
3
4
5
Perfectly
31
Q

A patient with HFrEF is currently not controlled on sacubutril/valsartan, ivabradine, empagliflozin. Which if the following do you recommend to add?

a. Spironolactone
b. Amiodarone
c. Ramipril
d. Atenolol

A

a. Spironolactone

32
Q

A 65 year old woman has been admitted to the coronary care unit with a left ventricular myocardial infarction. She develops acute decompensated heart failure with marked pulmonary edema.

Which of the following drugs would be the most useful?

a-digoxin
b-furosemide
c-minoxidil
d. propranolol

A

b-furosemide

33
Q

A 65 year old woman has been admitted to the coronary care unit with a left ventricular myocardial infarction. She develops acute decompensated heart failure with marked pulmonary edema.

Which of the following best describes why the pervious chosen drug is used to treat pulmonary edema?

a-It increases the force of myocardial contraction
b-Its powerful diuretic effect reduces pulmonary congestion
c-It blocks beta adrenoceptors
d-It has direct effect on gas exchange in the lung

A

b-Its powerful diuretic effect reduces pulmonary congestion

34
Q

The patient’s SBP was found to be 160 mmHg. Which of the following drugs should be added?

a- IV nitroglycerine
b- carvedilol
c- dobutamine
d-Diltiazem

A

a- IV nitroglycerine

35
Q

Which of the following medications could be used for this patient, and has natriuretic and vasodilating actions:

a- atenolol
b- digoxin
c- nesiritide
d- dopamine

A

c- nesiritide

36
Q

After stabilization of the patient’s condition, the patient is prepared for discharge. The following drug combination could be useful to the patient:

a- captopril+ valsartan
b- sacubitril/valsartan + spironolactone
c- sacubitril/valsartan + enalapril
d- propranolol + ivabradine

A

b- sacubitril/valsartan + spironolactone

37
Q

Which of the following is an absolute contraindication for ARNI therapy?

a- moderate renal impairment
b- moderate hepatic impairment
c- SBP> 140 mmHg
d- use of ACE inhibitor with the past 36 hr due to increase the risk of angioedema

A

d- use of ACE inhibitor with the past 36 hr due to increase the risk of angioedema

38
Q

A 70 year old man came to the emergency room with severe shortness of breath and tachycardia. He had a history of chronic heart failure. HIs condition was diagnose as cardiogenic (SBP was 80/50)

The following drug is essential for this patient

a- propranolol
b- dobutamine
c- nitroprusside
d- digoxin

A

b- dobutamine

39
Q

Which of the following statements regarding the action of inotropes is true?

a- Isoproterenol activation of alpha receptors may increase systemic blood pressure

b- Milrinone acts on beta-1 receptors to increase iontropy

c- The chronotropic effect of dobutamine is mediated through action on beta- 1 receptors

d- Amrinone is PDE IV inhibitor

A

c- The chronotropic effect of dobutamine is mediated through action on beta- 1 receptors

40
Q

Which of the following agents increase cardiac contractility through a mechanism other than an increase in intracellular cAMP?

a- levosimendan
b- dopamine
c- dobutamine
d- epinephrine

A

a- levosimendan

41
Q

A 50 year old post menopausal female patient with chronic hypertension had her blood pressure above target despite a daily regimen of lisinopril, hydrochlorothiazide and amlodipine. Patient is not known to ischemic heart or suffering from arrhythmia

What could be the further step to control this resistant hypertension?

a. Add an agent hydralazine
b. Initiate therapy with 25 mg of spironolactone if potassium levels permit
c. Add an ARB
d. Initiate therapy with bisoprolol

A

b. Initiate therapy with 25 mg of spironolactone if potassium levels permit

42
Q

If renal impairment is associated in this patient, which of the following ACE inhibitors does not need dose adjustment?

a. Lisinnopril
b. Fosinopril
c. Captopril
d. Enalapril

A

b. Fosinopril

43
Q

A 65 year old man with a 2 month history of progressive headache presented to the emergency department with nausea, vomiting, visual disturbance, and confusion for 1 day, his blood pressure was noted as 195/123 mm Hg, MRI of the brain was performed in the emergency department and demonstrated hypertensive encephalopathy.

Indicate the appropriate anti-hypertensive agent to be prescribed promptly to this patient

a. IV labetalol
b. Oral minoxidil
c. Oral prazosin
d. Oral alpha methyldopa

A

a. IV labetalol

44
Q

Which of the following anti-hypertensive medication can causes acidosis and arrhythmia in this elderly patient?

a. Nicardipine
b. Na nitroprusside
c. Nifedipine
d. Nitroglycerine

A

b. Na nitroprusside

45
Q

If the patient had developed acute pulmonary edema, what would be the appropriate diuretic to be added?

a. Frusemide
b. Thiazide
c. Spironolactone
d. Acetazolamide

A

a. Frusemide

46
Q

Two days later the patient’s blood pressure was set at 130/80 on an oral single pill combination of candesartan and hydrochlorothiazide

a. Hyperkalemia
b. Hypocalcemia
c. Hyperuricemia
d. Hypoglycemia

A

c. Hyperuricemia

47
Q

After three months, the patient came in your clinic complaining of recurrent chest pain on exertion, on checking lipid profile and ECG he was found to be dyslipidemic with established diagnosis of stable angina. What would be the preferred anti-hypertensive agent that most likely to be added?

a. Bisoprolol
b. Hydralazine
c. Lisinopril
d. Clonidine

A

a. Bisoprolol

48
Q

A 65 year old man is brought to the emergency department 30 min after the onset of right sided weakness and aphasia. Imaging studies ruled out cerebral hemorrhage as the cause of his acute symptoms of stroke.

Which if the following drugs would likely to be the most preferred to improve this patient’s clinical out come?

a. Alteplase
b. Factor VII
c. Streptokinase
d, Clopidogrel

A

a. Alteplase

49
Q

A 65 year old man is brought to the emergency department 30 min after the onset of right sided weakness and aphasia. Imaging studies ruled out cerebral hemorrhage as the cause of his acute symptoms of stroke.

Which of the following most accurately describes how the drug selected in the question 1 exerts it intended effect?

a. Blocks platelet ADP receptors
b. Inhibitors platelet thromboxane production
c. Inhibitors synthesis
d. Blocking glycoprotein IIb/IIIa platelets receptors
e. Promotes conversion of plasminogen to plasmin

A

e. Promotes conversion of plasminogen to plasmin

50
Q

Over the next 2 days, the patient’s symptoms resolved completely. To prevent a recurrence of this disease, the patient is most likely to be treated with which of the following?

a. Aminocaproic acid
b. Enoxaparin
c. Streptokinase
d. Aspirin
e. Lepirudin

A

d. Aspirin

51
Q

If the patient is unable to tolerate aspirin, which of the following drugs can be prescribed?

a. Abciximab
b. Alteplase
d. Streptokinase
e. Clopidogrel

A

e. Clopidogrel

52
Q

Which of the following most accurately describes the mechanism of eptifibatide anticlotting action?

A

a. Activation of antithrombin III
b. Blockade of post translational modification of clotting factors
c. Inhibition of thromboxane production
d. Irreversible inhibition of platelet ADP receptors
e. Reversible inhibition of glycoprotein IIb/IIIa receptors

53
Q

A 67 year old woman presents with pain in her left thigh muscle. Duplex ultrasonography indicates the presence of DVT in the affected limb.

The decision was made to treat this woman with enoxaparin. Relative to unfractionated heparin, enoxaparin..

a. Can be used without monitoring the patients a PTT
b. Has a shorter duration of action
c. Is more likely to be given intravenously
d. Is more likely to cause thrombosis and thrombocytopenia

A

a. Can be used without monitoring the patients a PTT

54
Q

During the next week the patient started on warfarin and her heparin was discontinued. Two months later, she returned with a sever nose bleed. This patient should be treated with which of the following?

a. Aminocaproic acid
b. Protamine sulphate
c. Vitamin K1
d. Factor VIII

A

c. Vitamin K1

55
Q

Which of the following correctly applies to the action of vitamin K

a. Acts as a cofactor for activation of some clotting factors
b. Acts to increase the transportation of mRNA for clotting factors
c. Is a hydrophilic molecule co-transported with glucose
d. Is used to treat pernicious anemia

A

a. Acts as a cofactor for activation of some clotting factors

56
Q

Which of the following is the most appropriate anticoagulant drug for parentral administration in a patient with deep vein thrombosis and has a history of heparin induced thrombocytopenia (HIT)

a. Rivaroxaban
b. Argatroban
c. Clopidogrel
d. Dabigatran

A

b. Argatroban

57
Q

Which of the following most accurately describes the mechanism of Argatroban

a. Rivaroxaban
b. Argatroban
c. Clopidogrel
d. Dabigatran

A

b. Argatroban

58
Q

Which of the following enhance the action of oral warfarin?

a. Rifampicin
b. Aspirin
c. Phenobarbitone
d. Carbamazepine

A

b. Aspirin

59
Q

Mr. AM sudden is a 65 year old gentleman who presented to the ER approximately one hour after a feeling of malaise and then fainted. Mr has AM has a history hypertension, diabetes and dyslipidemia.

What is the first line vasopressor drug to be used to treat the shock?

A

Norepinephrine

60
Q

Which medications below are used in cardiogenic shock and provide a positive inotropic effect on the heart

A. Nitroglycerin
B. Sodium nitroprusside
C. Dobutamine
D. Norepinephrine

A

C. Dobutamine

61
Q

An anti-choline esterase drug that is used in diagnosis of myasthenia crises is:

a) Physostigmine
b) Edrophonium
c) Atropine
d) Pyridostigmine

A

b) Edrophonium

62
Q

For long term symptomatic treatment of muscle weakness in MG patients the following drug is used

a) Acetylcholine
b) Edrophonium
c) Pyridostigmine
d) Propranolol

A

c) Pyridostigmine

63
Q

The drug chosen acts by the following mechanism

a) Blocks muscarinic receptors
b) Activates adrenergic receptors
c) Increase Ach at both N & M site
d) Non depolarizing neuromuscular blockade

A

c) Increase Ach at both N & M site

64
Q

The following drug is used as an adjuvant therapy to potentiate the action of anti-choline esterase?

a) Propranolol
b) Ephedrine
c) Quinidine
d) Curare

A

b) Ephedrine

65
Q

To decrease the unwanted muscarinic effects of neostigmine following drug may benefit

a) Ephedrine
b) Caffeine
c) Atropine
d) D-tubocurarine

A

c) Atropine

66
Q

The following drug should be avoided if possible in this patient

a) Atropine
b) Caffeine
c) Neostigmine
d) Aminoglycosides

A

d) Aminoglycosides

67
Q

The following is immunosuppressant to decrease Abs formation

a) Corticosteroids
b) Beta-blockers
c) Aminoglycosides
d) Adrenaline

A

a) Corticosteroids

68
Q

For refractory cases a new drug is approved for patients with generalized MG who are anti-AchR +ve

a) Edrophonium
b) Ephedrine
c) Succinyl choline
d) Eculizumab

A

d) Eculizumab

69
Q

The mechanism of action of triptans involves

a. Vasoconstriction
b. Vasodilatation
c. Analgesic
d. Antiemetic

A

a. Vasoconstriction

70
Q

A 65 year old man was seen at a clinic because of muscle weakness, emotional instability, burning of the tongue, and alternation constipation and diarrhea. Physical examination showed a pale man with beefy red tongue, loss of vibratory sense in the lower extremities and ataxia.

Which of the following drugs would be most appropriate for this patient? What is its route of administration?

a) Ferrous sulfate
b) Deferoxamine
c) Iron dextran
d) Cyanocobalamin

A

d) Cyanocobalamin, Intra muscular injection

71
Q

What is the course duration of this vitamin to correct the symptoms:

a) Daily intake for 2 weeks, then once per month
b) 3-6 months
c) 2 weeks
d) 10 days

A

a) Daily intake for 2 weeks, then once per month

72
Q

What is the mechanism of absorption of this vitamin in the terminal ilium?

a) Simple diffusion
b) Active transport
c) Filtration
d) Endocytosis

A

d) Endocytosis

73
Q

The factor needed for absorption of this element orally?

a) Transferrin
b) Intrinsic factor
c) Ferroprotein
d) Ferritin

A

b) Intrinsic factor

74
Q

A 34 year old woman was seen at a clinic because of severe weakness and dizziness for the past several months. The woman had a long history of menorrhagia and of chronic headaches for which she had been using several analgesic medication on a daily basis. Physical examination revealed a pale, lethargic female appearing older than her stated age.

Which of the following drugs would be most appropriate for this patient? Give the route of administration for this medication.

a) Cyanocobalamin
b) Folic acid
c) Eryhtopoietin
d) Iron dextran

A

Iron dextran; intra muscular injection

75
Q

Which of the following action most likely mediated the therapeutic effect of prescribe drug?

a) Replacement of stores that are severely depleted
b) Increase life span of RBCs
c) Stimulation of erythropoietin secretion by kidney
d) None of the above

A

a) Replacement of stores that are severely depleted

76
Q

Which of the following drugs would be most appropriate for her son who ingested several tablets of his mother’s medication and begins to suffer from two episodes of brownish vomit containing pill and hematemesis?

a) Deferoxamine
b) Folic acid
c) Ferrous sulfate
d) Erythropoietin

A

a) Deferoxamine